Recent content by mcintyre_ie

  1. M

    Proof Help - Rank of the transpose of a Matrix

    Thanks for your help. I've figured out the proof, using the dimension theorem as you suggested. It's not even that difficult once I know what result I'm moving towards.
  2. M

    Proof Help - Rank of the transpose of a Matrix

    Hi, I'm having trouble with a proof regarding the rank of the transpose of a matrix. Here's the question: Let A be an m x n matrix of rank r, which is of course less than or equal to min{m,n}. Prove that (A^t)A has the same rank as A. Where A^t = the transpose of A. I can easily...
  3. M

    How to Solve a Particle Motion Problem with Increasing Resistance?

    Thanks, that's really all I wanted to know.
  4. M

    How to Solve a Particle Motion Problem with Increasing Resistance?

    Yes, I know how to apply the force equation (F=Ma, etc), but what I don't understand is where the (8 + x/5) part comes from - I don't know how to firgure out that. That's my original question, and I still don't know how to do it.
  5. M

    How to Solve a Particle Motion Problem with Increasing Resistance?

    Ok, so now I'm just as lost. a = v(dv/dx), right? But I still don't know what I should be doing next...
  6. M

    How to Solve a Particle Motion Problem with Increasing Resistance?

    I’m still not getting it out… am I supposed to be using equations of motion to get a, which is equal to v(dv/dx) and (d^2x)/(dx^2)? I’ve been trying to do it that way, but I’m still getting nowhere. I still can't figure out where the 8m/s^2 and 9m/s^2 retardations come in. I’d be grateful for a...
  7. M

    How to Solve a Particle Motion Problem with Increasing Resistance?

    The way that I usually do it is to get a force equation for the question first... then work out dv/dx etc. Any help with actually getting a force equation?
  8. M

    How to Solve a Particle Motion Problem with Increasing Resistance?

    I’m having some difficulty with this question: (b) A particle starts with a speed of 20 m/s and moves in a straight line. The particle is subjected to a resistance which produces a retardation which is initially 8 m/s^2 and which increases uniformly with the distance moved, having a value of...
  9. M

    Help with Questions: Circles, Vectors, Wedges & dv/dt=g-kv

    I found it to be r^2 = 1/4 + 1/4 + 12. We don't have the innerproduct on our course, only dot product and scalar product, so it has to be solved using either or both of those (or other *basic* rules. Ok, I'll try it, but I'm not so sure about the lim part. Thanks for the help
  10. M

    Help with Questions: Circles, Vectors, Wedges & dv/dt=g-kv

    Hey, I’d appreciate some help with these questions: (c) Find the centre and radius of the circle x^2 + y^2 – x – y – 12 = 0. Find the equations of the tangent to this circle which are parallel to the line 7x – y = 0. Ok, so I found the line has a slope of 7, and the circle has a...
  11. M

    Help with Math Problems: Q.2(a), Q.6(c)!

    Ok, so I've tried it again... getting x=9, with a complex root when I sub that into get my two y values, which can't be right.
  12. M

    Help with Math Problems: Q.2(a), Q.6(c)!

    Um, I don't think that refers to my (current) question?
  13. M

    Analyzing Forces and Acceleration in a System of Connected Particles

    I was thinking that the 6kg mass would be the only mass to move downwards, but both the other mass and the pulley would move upwards? For some reason I was thinking that the pulley would have an acceleration of half of that of each of the masses.
  14. M

    Help with Math Problems: Q.2(a), Q.6(c)!

    Ok, so do you mean to solve the two equations simultaneously then? If that's what you mean, I just get an answer of "8x = 36"... Yes, I would also assume that "sole a system of equations" would mean the same thing throughout the english speaking world, however I am not a native english speaker...
  15. M

    Help with Math Problems: Q.2(a), Q.6(c)!

    Well yeah, but when you say "use your Algebra 2 skills" I'm just curious as to what I'm actually supposed to use...
Back
Top